LSAT and Law School Admissions Forum

Get expert LSAT preparation and law school admissions advice from PowerScore Test Preparation.

 Administrator
PowerScore Staff
  • PowerScore Staff
  • Posts: 8916
  • Joined: Feb 02, 2011
|
#85574
Complete Question Explanation

Flaw in the Reasoning—Cause and Effect. The correct answer choice is (B).

Answer choice (A):

Answer choice (B): This is the correct answer choice.

Answer choice (C):

Answer choice (D):

Answer choice (E):

This explanation is still in progress. Please post any questions below!
 studyhelp20
  • Posts: 28
  • Joined: Dec 09, 2020
|
#82048
Can I please receive some help with an explanation of the correct answer for this question? Along with explanations of the incorrect answers? Thanks
 Robert Carroll
PowerScore Staff
  • PowerScore Staff
  • Posts: 1787
  • Joined: Dec 06, 2013
|
#82166
study,

The premises in the stimulus establish correlations between two pairs:

chronic fatigue syndrome and lower-than-normal magnesium in the blood

malabsorption of magnesium by the digestive tract and some types of fatigue

The author concludes that a treatment to raise blood magnesium should help treat fatigue. Well, that assumes a causal relationship between blood magnesium levels and fatigue, and all the premises have given us is a correlation. The author seems to reason as follows: "Because low magnesium causes fatigue, treating people by raising magnesium should cure fatigue." The problem with the first half of that reasoning is that no premise HAS established that low magnesium causes fatigue - they are merely correlated. What if the low magnesium is a coincidence? What if fatigue causes low magnesium? Any of these possibilities, and more, would cast doubt on the author's reasoning.

The question is a Flaw in the Reasoning, so we're looking for a flaw here - maybe "failure to consider other causes of fatigue," "failure to consider that the correlation between fatigue and low magnesium is a coincidence," or "failure to consider that low magnesium may be the effect rather than a cause of fatigue."

Answer choice (A) seems to be saying that the author didn't prove that the correlation in the second sentence is perfect. This answer seems to be saying it's imperfect because low magnesium could be caused by other things besides malabsorption. But that wouldn't cast any doubt on the author's argument. Other causes of FATIGUE could.

Answer choice (B) claims that the mistake the author is making is in failing to consider that fatigue could lower magnesium levels - in other words, that the causal direction could be the reverse of what the author's argument relies on. This is certainly a flaw, and one we identified above. So we keep this answer.

Answer choice (C) is wrong because the fluctuation of magnesium is not at issue - we're taking about LOW magnesium, not just any variation in magnesium.

Answer choice (D) is wrong because the argument doesn't need to state the exact normal level in order to make its point, so the failure to do that is not a flaw.

Answer choice (E) is wrong because the argument does not need to specify the methods to raise magnesium - it suggested that raising magnesium is a good idea. How that's accomplished is really beside the point, so it's not a flaw to fail to specify.

Robert Carroll

Get the most out of your LSAT Prep Plus subscription.

Analyze and track your performance with our Testing and Analytics Package.